Discussion

Which one of the following must be true about any acceptable product code?
(A)The digit 1 appears in some position before the digit 2.
(B)...
(C)...
(D)...
(E)...
(F)...
*This question is included in LG Sample 1: Basic Game, question #2

The solution is

Posted: 05/19/2011 19:55
I wanted to know exactly why this was the right answer.
Posted: 05/20/2011 12:01
If you use the "Reduce" button twice, you'll see the following text:


Inferences:
1. Since the fifth digit of the code must be more than the third, the fifth digit cannot be zero.

2. The second digit must be twice the first, and the only numbers available for the code are 1, 2, 3, 4, 5. Therefore, you can infer that one of the following two scenarios will arise:


Scenario A:

1 2 __ __ __

In scenario A, the 3RD slot can be either 0 OR 3, and the 5TH slot can be either 3 OR 4.




Scenario B:

2 4 __ __ __


In scenario B, the 3RD slot can be either 0 OR 1, and the 5TH slot can be either 1 OR 3.


-----------------------------------


Go to the answer choices, and start going through the process of elimination:

Since scenario B is possible, you can rule out choice "A".

Look at scenario A. Is it possible for 3 to appear before 1? No it is not. Now look at scenario B. You could have 2, 4, 0, 3, 1. You can rule out choice "B".

Look at both scenarios. In each case, 2 MUST come before 3. Choice "C" is correct.


You can stop here. But if you need proof that choices "D" and "E" are wrong, consider:

The sequence 1, 2, 0, 4, 3 proves choice "D" is wrong.

The sequence 1, 2, 3, 0, 4 proves choice "E" is wrong.
Posted: 02/23/2012 22:51
I don't understand why this is the correct answer, and your explanation is not helpful at all. A and C were the answers I was left with but I knew that any acceptable code had to start with either 2,4 or 1,2 wouldn't that mean that A would be absolutely correct? It says that at some time 1 will appear before 2 and it is true at some point 1 will have to appear before 2.
Posted: 09/05/2012 17:55
Zakya, it appears you answer your own question. This problem asks which must be true for "any" product code. Since you've already stated that all product codes start only with 1,2 or 2,4. Since 2 will always be either the first or second digit, and the only time it is the second digit is when 1 is the first digit, in all acceptable scenarios, 3 must follow 2.
Posted: 09/07/2012 20:08
Thank you for posting, James Armstrong.
| Edit
Posted: 02/24/2012 17:06
I think both A and C are correct
Posted: 02/24/2012 17:16
Ok never mind I got it. C is possible but it wants to know "what must be true".
Image Not Available
Contributor
Posted: 02/24/2012 18:25
You got it, Jamie. Thanks for using the app. Don't forget to rate and comment about the app in the App Store. We need your support to keep this service going.
Posted: 05/10/2012 00:57
I am confused about the wording in the answer choices, maybe someone could clear this up for me. All of the answer choices say "x appears 'in the same position' before y'. But in the explanation provided, the digit 2 appears before the number 3 in both scenarios but in a different position in each of the two scenarios.



Btw this app is a great help. Thanks to all involved
Posted: 05/29/2013 10:23
James, I think you misread. Instead of "in the same position", it's "in some position".
Posted: 10/21/2012 10:27
I don't understand the answer.
Posted: 10/22/2012 13:23
Perla, did you try following the explanation? Also, read the explanation in the thread.
Posted: 12/27/2012 15:09
Whats confusing me here is the word "some.". Answer A, is techincally correct as 1 does appear before 2 in "some" codes...well yes, those that start with 1,2. Those are "some" of the codes. Am I reading this wrong?
Posted: 05/29/2013 10:25
Yes, you are reading it incorrectly, Jorge. Answer A is not technically correct. The key phrase is "must be true". The explanation provides shows that in scenario B, 2 appears before 1.
Posted: 05/29/2013 09:31
I was stumped by this question too and I really think the key phrase here is "must be true". This is a phrase used a lot in LSAT practice and test so I think it's important to understand what this actually means.

In this question, the third constraint tells us that the only options for the first two numbers are ...
1/2 _ _ ~0
2/4 _ _ ~0
Therefore, 3 "MUST" come at some point after the 2. As in answer A, like I originally chose, 1 CAN come before two in the order, but it doesn't HAVE TO (like we see in the second option above) and that is why answer A is incorrect. Hope this helps! :)
Posted: 05/29/2013 10:12
Thanks, Avin.
Posted: 06/05/2013 13:33
Why can't E be the answer? If you have the product code 2,4,0,3,1 the digit 4 appears in some position before the digit 3.
Posted: 06/05/2013 20:44
Kristen, you cannot justify a choice to be true based on one example when the question is asking if it MUST be true in all cases. Please follow the discussion thread as this question has been analyzed thoroughly.
Posted: 09/02/2014 08:29
I was left with C and E and they both seemed right to me. Why was C correct?
Posted: 09/02/2014 13:27
Elizabeth, this problem has been analyzed several times previously. Please look through the thread. The key phrase you should pay attention to is "must be true".

You need to be signed in to perform that action.

Sign In